L2-limit of sum similar to the Itô integral

brownian motionstochastic-analysisstochastic-calculusstochastic-integrals

We define a sequence of partitions $\Pi_{n} = \{0=t_{0} < t_{1} <\ldots<t_{k_{n}} =T\}$ for $n\geq1.$
For any $\lambda \in[0,1]$ we set $t^{\lambda}_{n,j}=\lambda t_{j}+(1-\lambda)t_{j-1}.$
Then in $L^2$ $$\lim_{n\to\infty} \sum_{j=1}^{k_{n}} W_{t^{\lambda}_{n,j}} \left(W_{t_{j}}-W_{t_{j-1}}\right) = \int_{0}^{T} W_{t}\,dW_{t}+\lambda T,$$
where $\int_{0}^{T}W_{t}\,dW_{t}$ is the Itô integral and $W_t$ the Brownian motion.

I do not know where to start to show this limit. Clearly for $\lambda=0$ we get the usual definition of the Itô integral. I tried to consider $(W_{t^{\lambda}_{n,j}}-W_{t_{j-1}}+W_{t_{j-1}})$, but I could not solve that either. Any idea?

Best Answer

The trick is to rewrite the sum to

$$\sum_{j=1}^{k_{n}} W_{t_{j-1}} \left(W_{t_{j}}-W_{t_{j-1}}\right) +\sum_{j=1}^{k_{n}} (W_{t^{\lambda}_{n,j}}-W_{t_{j-1}})^{2} + \sum_{j=1}^{k_{n}} (W_{t^{\lambda}_{n,j}}-W_{t_{j-1}}) \left(W_{t_{j}}-W_{t^{\lambda}_{n,j}}\right).$$

Then the first sum is clear, the second is almost a quadratic variation (gives $\lambda T$) and the last one goes to 0.